From c8714b337d30b2e8a752ae011c59d67832e24c9f Mon Sep 17 00:00:00 2001 From: wieerwill Date: Sat, 26 Mar 2022 17:27:58 +0100 Subject: [PATCH] =?UTF-8?q?L=C3=B6sungen=201+2?= MIME-Version: 1.0 Content-Type: text/plain; charset=UTF-8 Content-Transfer-Encoding: 8bit --- ...kprogrammierung - Prüfungsvorbereitung.pdf | 4 +- ...kprogrammierung - Prüfungsvorbereitung.tex | 37 +++++++++++++++++++ 2 files changed, 39 insertions(+), 2 deletions(-) diff --git a/Logik und Logikprogrammierung - Prüfungsvorbereitung.pdf b/Logik und Logikprogrammierung - Prüfungsvorbereitung.pdf index 73a0178..262830c 100644 --- a/Logik und Logikprogrammierung - Prüfungsvorbereitung.pdf +++ b/Logik und Logikprogrammierung - Prüfungsvorbereitung.pdf @@ -1,3 +1,3 @@ version https://git-lfs.github.com/spec/v1 -oid sha256:d4faefd2ccceab636ef89f2ec757f7df932ceafa5c56592a0c7e8ed45e189f11 -size 162920 +oid sha256:254abb252c523be3686d05ec3a5dd23a3b4a20681afa37eb6c724fceb5e1f9d9 +size 244234 diff --git a/Logik und Logikprogrammierung - Prüfungsvorbereitung.tex b/Logik und Logikprogrammierung - Prüfungsvorbereitung.tex index 1c839f0..503767f 100644 --- a/Logik und Logikprogrammierung - Prüfungsvorbereitung.tex +++ b/Logik und Logikprogrammierung - Prüfungsvorbereitung.tex @@ -26,6 +26,10 @@ \SolutionEmphasis{\small} \geometry{top=1cm,left=1cm,right=1cm,bottom=1cm} +\usepackage{pifont} +\newcommand{\cmark}{\ding{51}} +\newcommand{\xmark}{\ding{55}} + \pdfinfo{ /Title (Logik und Logikprogrammierung - Prüfungsvorbereitung) /Creator (TeX) @@ -98,22 +102,33 @@ \begin{parts} \part Der Korrektheitssatz der Aussagenlogik für den Wahrheitswertebereich $B$ lautet... \begin{solution} + Für jede Menge von Formeln $\Gamma$ und jede Formel $\varphi$ gilt $\Gamma\vdash\varphi\Rightarrow\Gamma\vdash_B\varphi$. \end{solution} \part Eine Menge von Formeln $\Gamma$ heißt erfüllbar, wenn... \begin{solution} + Sei $\Gamma$ eine Menge von Formeln. $\Gamma$ heißt erfüllbar, wenn es eine passende B-Belegung $B$ gibt mit $B(\gamma) = 1_B$ für alle $\gamma\in\Gamma$. + \end{solution} + + \part Der Satz von Cook lautet... + \begin{solution} + Die Erfüllbarkeit einer endlichen Menge $\Gamma$ ist NP-vollständig. \end{solution} \part Zwei Formeln $\alpha$ und $\beta$ heißen äquivalent, wenn... \begin{solution} + Zwei Formeln $\alpha$ und $\beta$ heißen äquivalent $(\alpha\equiv\beta)$, wenn für alle passenden B-Belegungen $B$ gilt: $B(\alpha) =B(\beta)$. \end{solution} \part Der Kompaktheitssatz der Aussagenlogik lautet... \begin{solution} + Sei $\Gamma$ eine u.U. unendliche Menge von Formeln. Dann gilt $\Gamma$ unerfüllbar $\Leftarrow\Rightarrow\exists\Gamma'\subseteq\Gamma$ endlich: $\Gamma'$ unerfüllbar \end{solution} \part Eine Horn Klausel ist eine Formel der Form \begin{solution} + Eine Hornklausel hat die Form $(\lnot\bot\wedge p_1\wedge p_2\wedge ... \wedge p_n)\rightarrow q$ für $n\geq 0$, atomare Formeln $p_1 ,p_2 ,... ,p_n$ und $q$ atomare Formel oder $q=\bot$. + Eine Hornformel ist eine Konjunktion von Hornklauseln. \end{solution} \end{parts} @@ -121,14 +136,36 @@ \begin{parts} \part Werte die Formel $\varpi_a=\lnot p \wedge \lnot\lnot p$ im Heytingschen Wahrheitswertebereich $H_{\mathbb{R}}$ aus für die $H_{\mathbb{R}}$-Belegung $B$ mit $B(p)=\mathbb{R}\backslash \{0\}$ \begin{solution} + $B_{H_{mathbb{R}}}(\lnot p \wedge \lnot\lnot p)= Inneres(\mathbb{R}/ p)\cap p= 1$ \end{solution} \part Überprüfe ob die Formel $\varphi_B=(\lnot p\rightarrow \lnot p)\rightarrow p$ eine $K_3$-Tautologie ist. Ist $\varphi_b$ eine $B_{\mathbb{R}}$ Tautologie? \begin{solution} + + \begin{tabular}{c|c|c|c} + $p$ & $\lnot p$ & $\phi=(\lnot p\rightarrow \lnot p)$ & $\phi\rightarrow p$ \\\hline + 0 & 1 & 1 & 0 \\ + $\frac{1}{2}$ & $\frac{1}{2}$ & 1 & $\frac{1}{2}$ \\ + 1 & 0 & 1 & 1 \\ + \end{tabular} \end{solution} \part Überprüfe ob die semantische Folgeung $\{p\rightarrow q, q\rightarrow r\}\Vdash_B r\rightarrow\lnot p$ gilt. \begin{solution} + + \begin{tabular}{c|c|c|c|c|c|c|c} + $p$ & $q$ & $r$ & $\lnot p$ & $\Gamma_1=p\rightarrow q$ & $\Gamma_2=q\rightarrow r$ & $\Phi=r\rightarrow\lnot p$ & $\Gamma\Vdash\Phi$ \\\hline + 0 & 0 & 0 & 1 & 1 & 1 & 1 & \cmark \\ + 0 & 0 & 1 & 1 & 1 & 1 & 1 & \cmark \\ + 0 & 1 & 0 & 1 & 1 & 0 & 1 & \\ + 0 & 1 & 1 & 1 & 1 & 1 & 1 & \cmark \\ + 1 & 0 & 0 & 0 & 0 & 1 & 1 & \\ + 1 & 0 & 1 & 0 & 0 & 1 & 0 & \\ + 1 & 1 & 0 & 0 & 1 & 0 & 1 & \\ + 1 & 1 & 1 & 0 & 1 & 1 & 0 & \xmark + \end{tabular} + + Folgerung gilt nicht \end{solution} \end{parts}